You are on page 1of 3

Classical Mechanics

Solution Set 12 Due: 20 November 2013 45. Prove the Jacobi identity for Poisson brackets: {f, {g, h}} + {h, {f, g }} + {g, {h, f }} = 0. (1)

by direct calculation. Expanding out the double brackets you will nd 24 distinct terms which should cancel in pairs. Solution: Let us write out the rst triple bracket: {f, {g, h}} =
k

f qk pk f qk

l 2

h g g h ql pl ql pl
2

f pk qk
2

h g g h ql pl ql pl

=
k

g h h g g h 2 h g + pk ql pl ql pk pl ql pk pl pk ql pl 2 g h h 2 g g 2 h 2 h g + qk ql pl ql qk pl ql qk pl qk ql pl h f f g + pl qk pl qk h f 2 g h f 2 g ql qk pk pl pl pk qk ql

f pk 2g pk ql

=
kl

kl

g f f g + pk ql pk ql

2 h g f g f 2 h 2h (2) qk pl qk ql pl pk ql qk pk pl

Notice that the terms involving second derivatives of g are symmetric under f h. The second triple bracket on the left of the Jacobi identity can be written {h, {f, g }} = {h, {g, f }}, so it will contribute second derivatives of g which are opposite in sign. The third triple bracket has no second derivatives of g , so we conclude that the left side of the Jacobi identity contributes no second derivatives of g . A parallel argument shows that it contributes no second derivatives of f or h either. Since every term has a second derivative of f , g , or h, we conclude that the left side is zero so the Jacobi identity holds. 46. As we know the total momentum k pk and total angular momentum J = k r k pk are important physical properties of any closed system. For the case of a single particle evaluate all of the Poisson brackets of px , py , pz , Jx , Jy , Jz with each other. Start with the Poisson brackets for the canonical variables r , p and exploit the algebraic identities satised by the P.B.s. Use the antisymmetric symbol klm to present your nal answers in compact form.

Solution: The fundamental Poisson brackets are {rk , rl } = {pk , pl } = 0 and {rk , pl } = kl . We can also write Jk = klm rl pm . Then {pj , Jk } = klm {pj , rl pm } = klm ({pj , rl }pm + rl {pj , pm }) = kjm pm = jkm pm {rj , Jk } = klm {rj , rl pm } = klm ({rj , rl }pm + rl {rj , pm }) = klj rl = jkl rl {Jj , Jk } = klm {Jj , rl pm } = klm ({Jj , rl }pm + rl {Jj , pm }) = klm ljn rn pm klm mjn pn rl = (kj mn kn jm )rn pm (kj ln kn lj )pn rl = rj pk rk pj = jkm mns rn ps = jkm Jm (3) The desired Poisson Brackets are then {pk , pl } = 0, {pk , Jl } = klm pm , {Jk , Jl } = klm Jm (4)

47. Spatial translations and rotation of the coordinates and momenta of a single particle are canonical transformations. Find the generating function for translations and for rotations and show that the innitesimal generator of a translation is a linear combination of momentum components and the innitesimal generator of a rotation is a linear combination of the components of angular momentum. Solution: Translations are the canonical transformation Qk = qk + ak , Pk = pk . The generating function for this is simply F2 = (ql + ak )Pk = qk Pk + ak Pk . Taking a innitesimal shows us that the innitesimal generator is G = ak pk , obviously a linear combination of the components of momentum. Rotations are described by an orthogonal matrix R, RRT = RT R = I , as follows Pk = Rkl pl , Qk = Rkl ql (5)

We seek an F2 (q, P ) type generating function of the form F2 = qk Mkl Pl , pk = Mkl Pl ,


T Qk = ql Mlk = Mkl ql

(6)

Comparing with the desired transformation we see that M T = R so that M = RT = R1 . T With this choice p = R1 P or P = Rp and Q = Rq as desired. Thus F2 = qk Rkl Pl . A T T rotation close to the identity is a matrix R = I + J where RR = I implies J = J to rst order in J . Then F2 qk Pk + q J T P = qk Pk q J P 1 1 G = q J p = qk pl Jkl = Jkl klm mij qi pj = Jkl klm Jm 2 2 where Jm are the components of the angular momentum. 48. Another important canonical transformation is a Galilei boost: For a single free particle it is r r + V t, p p + mV . 2

(7)

a) Find the generating function for this canonical transformation for nite V . Solution: We guess an F2 = (r + V t) P + aV r + bt. Then it would follow that R = F2 = r + V t, P p= F2 = P + aV r (8)

Comparing to the desired transformation we see that choosing a = m does the H = trick. The constant b can be determined from the dierence b + V P = H 2 2 V P mV /2 or b = mV /2. Thus F2 = (r + V t) P mV r mV 2 t 2 (9)

b) By taking V small determine the innitesimal generator K of Galilei transformations. Solution: We easily see that the innitesimal generator G = V (pt mr ). So we can identify three independent components K = pt mr . c) Show that K is a constant of the motion, even though its Poisson bracket with the Hamiltonian, which you should determine, is not zero. Solution: We have the general formula = {K , H } + K = 1 {K , p2 } + p = 1 pk {K , pk } + p = p + p = 0 (10) K t 2m m In the process we have shown that {K , H } = p = 0. K is a constant because of its explicit time dependence. d) Find the Poisson brackets of the components Kk with each other. Solution: We simply calculate {Kk , Kl } = {pk t, mrl } + {mrk , pl t} = +mtkl mtkl = 0 (11)

Although this wasnt asked, we can display the full Galilei Poisson bracket algebra including rotations Jk , boosts Kk , translations in space pk and translations in time H = p2 /2m: {pk , pl } = 0, {pk , H } = 0, {H, Jk } = 0, {H, Kk } = pk , {Jk , Jl } = klm Jm {pk , Jl } = klm pm , {Kk , Kl } = 0, {pk , Kl } = mkl {Kk , Jl } = klm Km (12)

You might also like